All Questions

Filter by
Sorted by
Tagged with
1 vote
0 answers
48 views

Derive a regular conditional distribution of $\beta_0 + \beta_1 X + \epsilon$ given $X$, when $\epsilon \sim N(0, \sigma^2)$

Given two random variables $X $ and $\epsilon$, where $\epsilon \sim N(0,\sigma)$, I would like to derive a conditional distribution of $\beta_0 + \beta_1 X + \epsilon$ given $X$. If it is necessary ...
MrFranzén's user avatar
1 vote
0 answers
93 views

If $G$ is a abelian with $|G|=p^n$ and $a$ is an element of maximum order in $G$, then $G$ is of the form $\langle a\rangle \times K.$

Prove that if $G$ is an abelian group of prime-power order $p^n$ and let $a$ be an element of maximum order in $G$, then $G$ can be written in the form $\langle a\rangle \times K.$ Our teacher told ...
ting730's user avatar
  • 11
3 votes
2 answers
647 views

Is it true that if $f$ is surjective from $A$ to $B$ then there is an injective function $g$ from $B$ to $A$? [duplicate]

I think the answer is yes because otherwise $f$ wouldn't be a function. Is this correct? And how would the formal proof go?
yolo expectz's user avatar
0 votes
0 answers
25 views

Prove the existence of a regular tetrahedron with some costraints on the center of mass

"Prove there exists in $\mathbb{R}^{3}$ a regular tetrahedron of real side $d$ such that if one puts in its vertices four strictly positive masses $m_1>m_2>m_3>m_4$ the center of mass is in $(...
Eugenio Di Paola's user avatar
1 vote
1 answer
139 views

INTEGRATION-type problem-Find the value of $\int_0^8f(t)dt$

If the function $f:[0,8] → R$ is differentiable then for $0<\alpha$, $\beta<2$ , $\int_0^8f(t)dt$ is equal to (A) $3[\alpha^3f(\alpha^2)+\beta^3f(\beta^2)]$ (B) $3[\alpha^3f(\alpha^2)+\beta^...
Samar Imam Zaidi's user avatar
1 vote
1 answer
66 views

comparing averages versus comparing the averages of percentage

Let’s say there are three rooms with 40 identical boxes in each room. Each box contains some number of cards inside. The number of cards in each box is different. It may be as many as 60 cards in one ...
brilliant's user avatar
  • 818
2 votes
1 answer
130 views

Quaternion algebra: $a \equiv 3$ or $5\mod 8$ implies $(a,2)_{\mathbb{Q}}$ is a division ring.

Question: let $a\in\mathbb{Z}$ such that $a\equiv 3$ or $5\mod 8$. Proof that $(a,2)_{\mathbb{Q}}$ is a division ring. Definition: a quaternion algebra over a field $F$ is a ring that is a $4$-...
Math's user avatar
  • 824
1 vote
1 answer
166 views

Solution to non-homogenous second-order differential equation from Navier-Stokes equation.

I was using the simplified Navier-Stokes equations to find the velocity profile within a cylindrical pipe, where I got the equation as: $ \alpha = \frac{\partial V^2}{\partial r^2}+\frac{1}{r}\frac{\...
Mughees Asif's user avatar
2 votes
1 answer
154 views

Does there exist an initial arrangement of 10 black squares such that all the squares will ultimately be black?

Let there be a $12×12$ table of white squares. We draw $10$ squares in black. If a white square has $2$ black neighbours, then we draw it in black. We say that $2$ squares are neighbours if they have ...
Ana's user avatar
  • 587
0 votes
1 answer
25 views

One More Ordinary Differential Equation

While working on some variations of a physics problem (how does the normal force vary in a circular loop with a constant attrition coefficient with regards to the angle ) , I arrived at the ...
RicardoMM's user avatar
  • 796
2 votes
1 answer
84 views

Calculus II: Limit exercise

I'm currently studying for my Calc exam and I came across this exercise. The problem is to find the values for $a,b,c \in \mathbb{R}$ so that the following limit exists: $$ \lim_{(x,y) \rightarrow (...
Dabruh's user avatar
  • 111
-1 votes
2 answers
72 views

Solve the equation $3x^2+5x \equiv -1357 \pmod{7919}$

Solve the modular equation $3x^2+5x \equiv -1357 \pmod{7919}$. Is there anybody help me with this ? I have thought several times but not to find out the solution. Thank you.
Nguyen Thy's user avatar
1 vote
1 answer
69 views

Let $G$ be a group and $c\in G$. Define $*$ on $G$ by: $x*y=xdy$ for all $x,y\in G$, where $d$ is the inverse of $c$. Prove $(G,*)$ is a group.

Let $G$ be a group and $c$ be a fixed element of $G$. Define a new operation $*$ on G by: $$x*y=xdy$$ for all $x$ and $y$ in $G$, where $d$ is the inverse of $c$. Prove that $G$ is a group under ...
user avatar
0 votes
2 answers
91 views

Sum of Legendre sequence $S=\sum_{x=0}^{p-1}\left(\frac{x(x+k)}{p}\right)$

Calculate the sum of Legendre sequence $S(x):=\displaystyle \sum_{x=0}^{p-1}\left(\dfrac{x(x+k)}{p}\right)$ with $p > 3$ prime number, $k \in \mathbb{N}$ and $\text{gcd}(p,k)=1$. I have tried to ...
Nguyen Thy's user avatar
-1 votes
2 answers
28 views

squareroot of x^2 in limits infinity to -infinity...

$\frac{x}{a^2\sqrt{x^2+a^2}}\bigg|^{\infty}_{-\infty} = ?$ $=\Bigg[\frac{x}{a^2\sqrt{(x^2)(1+\frac{a^2}{x^2})}}\Bigg]^{\infty}_{-\infty}$ $=\Bigg[\frac{x}{a^2\sqrt{(x^2)}\sqrt{(1+\frac{a^2}{x^2})}}\...
pico's user avatar
  • 951
0 votes
1 answer
272 views

Example of a dense set which is not nowhere dense

I can't find an example of a dense set A in $\mathbb{R}^{2}$ which is not nowhere dense. The definition of nowhere dense set given in the exercise is as follows: Let $\mathrm{X}$ be a topological ...
Laura Gigliotti's user avatar
-1 votes
2 answers
44 views

calculating some limits with elementary ways

How the following limits can be computed without using Taylor series, Laurent series, or L'Hospital? I) $$\lim_{x\to\pi/4} \frac{\ln\left(\tan\left(x\right)\right)}{\cos\left(2x\right)}$$ II) $$\...
Absurd's user avatar
  • 369
0 votes
2 answers
172 views

why commutative integral with limit is important in real analysis? [closed]

why commutative integral with limit is important in real analysis ? Why $\lim_{n\to\infty }\int f_n=\int \lim_{n\to\infty } f_n $ is important ?
amir bahadory's user avatar
1 vote
2 answers
98 views

Open subset Proof

Let $U\subseteq\mathbb{R}^2$ be defined as $\{$ $(x_1,x_2) \in \Bbb R^2$ $:$ $x_1>0$ $\}$. Then $U$ is open. My proof: Let $\textbf{x} \in U$ then $\textbf{x}= (x,y)$ where $x>0$. Let $r= x$. ...
user avatar
0 votes
2 answers
162 views

Monotonicity of a function without using differentiation

I want to prove the monotonicity of the following function WITHOUT using derivatives. f(x)=x²-4x+3, on the interval [-1,1] let x1 be less than x2 square both sides x1² less than x2² add -4x to ...
Cuenc's user avatar
  • 11
0 votes
0 answers
125 views

Parabola - Converting Cartesian Coordinates to Polar Coordinates

Just a simple parabola, $ y = x^2 $ . So, doing the maths: $r sin \theta = r^2 (cos(\theta))^2$. Then, I have $ r = \frac{sin(\theta)}{(cos(\theta))^2}$ . I'm interested when $ \frac{3\pi}{4} < ...
O Rapaz's user avatar
  • 27
1 vote
1 answer
434 views

Geometry question with perpendicular lines and angle bisectors in triangle

For any triangle ABC 1) Draw angle bisector of angle BAC. Draw perpendicular bisector of BC. Mark the intersection point as D. 2) Draw altitudes from D to AB and AC. (Extend line as necessary). ...
Gareth Ma's user avatar
  • 1,853
1 vote
3 answers
2k views

Solutions for problem 1-1 of Introduction to Algorithms, Third Edition, by CLRS

I'm working problem 1-1 of the textbook Introduction to Algorithms, Third Edition, by CLRS, and need to solve $n\log(n) = 10^6$, where the logarithm is base $2$. My first thought was to use the ...
The Pointer's user avatar
  • 4,192
0 votes
0 answers
49 views

Can we define sum over an uncountable set? [duplicate]

Is it possible by some means to define a notion of sum over the elements of an uncountable set $S$ of real numbers.I thought of something like $\sum_{\alpha \in \tau } a_\alpha:=\sup_{T\subset S ,|T|=...
Kishalay Sarkar's user avatar
3 votes
1 answer
2k views

When does equality hold in the triangle inequality?

We consider the supremum norm $\|f\|=\sup_{x\in [a,b]} |f(x)|$ in the space $B([a,b],\mathbb C)$ of all bounded functions $f: [a,b] \rightarrow \mathbb C$. We obviously have in general that $\|f+g\|...
Alex's user avatar
  • 2,191
-1 votes
1 answer
29 views

Can Someone Help me solving this equation using modulus operation? [closed]

X mod(35-3*X)=0 How to find values of X?
Kartik Bhatia's user avatar
2 votes
2 answers
75 views

What is $\frac{\partial^2(g\circ f)}{\partial x_i \partial x_j}(c)$

If $f:A\to \mathbb R^k$ and $g:B\to \mathbb R$ are two functions of class $C^2$ and their composition is well defined. For $c \in A$ what is $$\frac{\partial^2(g\circ f)}{\partial x_i \partial x_j}(...
Nasal's user avatar
  • 798
0 votes
1 answer
890 views

Sum of "positive" numbers

Given this definition of a positive real number: $$\forall x\in\mathbb{R},x\geq 0:\Leftrightarrow\exists y\in\mathbb{R},x=y^2$$ how to prove that the sum of two positive numbers is a positive number? ...
Maxence1402's user avatar
0 votes
1 answer
201 views

Distributive property conjunction / existential quantifier

Given the following predicate; $(\exists x:X.P(x)\implies Q(x)) \wedge \forall y:X.P(y)$ Applying the distributive property of $\wedge$ / $\exists$ $\equiv \exists x:X.(P(x)\implies Q(x)) \wedge \...
Sophia's user avatar
  • 3
2 votes
1 answer
625 views

Area of bounded region

$A$ is the region of the complex plane $\{z: z/4$ and $4/z'$ have real and imaginary part in $(0, 1)\}$, then $[p]$(where $p$ is the area of the region $A$ and $[.]$ denotes the greatest integer ...
Equation_Charmer's user avatar
1 vote
1 answer
75 views

Why is $\sum_{p\leq x} \log p=O(x)$?

I know that $$2^{2n}=(1+1)^{2n}={2n\choose 0}+\cdots+{2n\choose n}+\cdots+{2n\choose 2n}\geq {2n\choose n}\geq \prod_{n < p \leq 2n} p$$ If we let $n=2^{k-1}$ this implies that $\prod_{2^{k-1} <...
Jhon Doe's user avatar
  • 2,912
5 votes
4 answers
335 views

Understanding how contrapositive work

I want to understand contrapositive clearly. I'll start by saying: "If it is sunny, then there is light". The statement is true. But now consider the contrapositive: " If there is no light then it is ...
stopwatchingmesleep's user avatar
2 votes
2 answers
87 views

Prove $\{(2,0)\}$ is not extendable to a basis for $\mathbb{Z}^2$

I want to show that not every independent set in a free module is extendable to a basis. Let $R=\mathbb{Z}$ and consider the $R$-module $M=R^2=\mathbb{Z}^2$. Then, $M$ is free of rank $2$ and $S=\{(2,...
MyWorld's user avatar
  • 2,398
2 votes
1 answer
166 views

Creating formula from algorithm

I have this algorithm which i have to convert to polynom of degree 2: Algorithm is : ...
Patrik Bašo's user avatar
0 votes
0 answers
69 views

Example of Herbrand quotient hexagon

I want to construct an example of Herbrand quotient's hexagon diagram. Let $0 \to p\mathbb{Z} \to \mathbb{Z} \to \mathbb{Z}/p\mathbb{Z} \to 0$ be a short exact sequence of $\mathbb{Z}$-modules and we ...
user avatar
1 vote
0 answers
69 views

A guide on using integration by substitution.

Integration by substitution is a tricky thing to master, I'm learning it from books and at some place my book has done something like this $$\int \csc\theta \cot\theta \; d\theta = ?$$ $$ \int \...
Knight wants Loong back's user avatar
0 votes
2 answers
153 views

Prove:$\lim_{x\to \infty}\frac{\lfloor 3e^x\rfloor+2}{\lfloor 2e^x\rfloor+1}=\frac{3}{2}$

Prove that: $\displaystyle\lim_{x\to \infty}\frac{\lfloor 3e^x\rfloor+2}{\lfloor 2e^x\rfloor+1}=\frac{3}{2}$ My attempt: Let $f:\mathbb R\to S\subset\mathbb Q$ $$f(x)=\frac{\lfloor 3e^x\rfloor+...
PinkyWay's user avatar
  • 4,575
3 votes
0 answers
75 views

Morita-invertible von Neumann algebras

I am familiar with the Morita theory of rings, and the hermitian Morita theory of rings with involution, and I am trying to understand some parallels and differences with the Morita theory of von ...
Captain Lama's user avatar
  • 25.8k
0 votes
3 answers
1k views

Monty Hall Problem - $4$ doors where you pick one by one

I encountered the problem of Monty Hall Problem with $4$ doors ($3$ doors $1$ goat). However in this variation you pick an initial door. Monty reveals one goat. Thereafter you either switch or stay - ...
Mats Gubaboo's user avatar
0 votes
2 answers
59 views

Find the maximal value of $\mu^{-1}\cos\theta+\sin\theta$

This is part of a physics problem I was doing yesterday. I am supposed to find the maximum value of $$\mu^{-1}\cos\theta+\sin\theta$$ This is is supposed to produce the result of $\sqrt{1+\mu^{-2}}$. ...
Physics_Learner's user avatar
0 votes
2 answers
1k views

Proving a bipartite graph is not planar

I have drawn the bipartite graph K3,3. And I need to prove this graph isn't planar. I used contradiction, assuming it was planar, using Eulers formula to prove it wasn't planar. I concluded my graph ...
QuestionAnswer13's user avatar
0 votes
0 answers
48 views

The gradient of $\| x - \alpha \nabla f(x) \|_2^2$ with respect to $x$?

The gradient of $\| x - \alpha \nabla f(x) \|_2^2$ with respect to $x$, where $x \in \mathbb{C}^n$, $\nabla f(x) \in \mathbb{C}^n$, and $\alpha \in \mathbb{R}$? Following greg's comments, please see ...
learning's user avatar
  • 661
1 vote
0 answers
225 views

The wedge product of covectors

On a textbook, the wedge product of the covectors $\Omega$ is spanned on basis vectors $dx$ using this notation. $$\Omega^i=\Omega^i_\mu dx^{\mu}$$ $i$ is the name of the covector, not an index. $$\...
Tursinbay's user avatar
  • 307
1 vote
1 answer
3k views

Finding a Sine Wave That Goes Through a Set of Given Points

This question is coming from a thought experiment I was having by myself. Let's say I have 3 random points on a x-y plane (with different x values). Let them be (1,3), (2,5), (5, 1). Is there a sine ...
ThunderSea's user avatar
1 vote
1 answer
201 views

finding the volume of a tethrahedron (course MIT 18.02 multivariate calculus)

The exercice of the session 48 of the course MIT 18.02 multivariate calculus, version 2010, obtainable here reads as follows My question: While I get the derivation and computation, what I'm ...
ecjb's user avatar
  • 995
-1 votes
1 answer
83 views

Probability of drawing of $5$ cards from a deck

What is the probability of drawing of $5$ cards from a deck and getting 1) $3$ spades 2) at most $2$ spades 3) at least $3$ spades ? My attempt is: $(13C3)(49C2)/(52C5)$ $(13C0)(52C5)+(13C1)(...
LaTeXKhan's user avatar
1 vote
2 answers
111 views

In a convex pentagon ABCDE: $AB=AC$, $AE=AD$, $\angle CAD= \angle ABE + \angle AEB$, M is the midpoint of BE. Prove $2AM=CD$

I'd love to know how to prove this. I don't exactly know how to begin. I can recognize the two isosceles triangles ABC and ADE. I can also express M as $\frac{B+E}2$. But then I'm stuck...
aradarbel10's user avatar
1 vote
1 answer
136 views

Limit of polynomial functions

Problem: Given a sequence $(f_n)_{n\in\mathbb{N}}$ of polynomial functions such that $f_n(x)=\sum_{j=0}^{n}\dfrac{j^3}{4^{j+2}}x^j$, find the largest real number $r$, such that $(f_n)_{n\in\mathbb{N}}$...
user avatar
0 votes
1 answer
631 views

Calculate sum of sum using Z transform

$$S = \sum_{n=0}^{\infty} \frac{1}{3^n} \sum_{k=0}^{n} \frac{k}{2^k}$$ We know that if one signal y can be written as $$y(n) = \sum_{k=0}^{n} x(k)$$ then it's Z transform is $$ \frac{z}{z-1} X(z) $$ ...
SADBOYS's user avatar
  • 1,219
2 votes
0 answers
225 views

Equivalence of two definitions of cross product for cohomology

Allen Hatcher's book gives two versions of the cross product. The first one is: \begin{eqnarray} a\times b\equiv p_1^*(a)\cup p_2^*(b), \end{eqnarray} in which $p_1:X\times Y\rightarrow X$, $p_2:X\...
Smart Yao's user avatar
  • 574

15 30 50 per page